Stay ahead of learning milestones! Enroll in a class over the summer!

G
Topic
First Poster
Last Poster
k a May Highlights and 2025 AoPS Online Class Information
jlacosta   0
May 1, 2025
May is an exciting month! National MATHCOUNTS is the second week of May in Washington D.C. and our Founder, Richard Rusczyk will be presenting a seminar, Preparing Strong Math Students for College and Careers, on May 11th.

Are you interested in working towards MATHCOUNTS and don’t know where to start? We have you covered! If you have taken Prealgebra, then you are ready for MATHCOUNTS/AMC 8 Basics. Already aiming for State or National MATHCOUNTS and harder AMC 8 problems? Then our MATHCOUNTS/AMC 8 Advanced course is for you.

Summer camps are starting next month at the Virtual Campus in math and language arts that are 2 - to 4 - weeks in duration. Spaces are still available - don’t miss your chance to have an enriching summer experience. There are middle and high school competition math camps as well as Math Beasts camps that review key topics coupled with fun explorations covering areas such as graph theory (Math Beasts Camp 6), cryptography (Math Beasts Camp 7-8), and topology (Math Beasts Camp 8-9)!

Be sure to mark your calendars for the following upcoming events:
[list][*]May 9th, 4:30pm PT/7:30pm ET, Casework 2: Overwhelming Evidence — A Text Adventure, a game where participants will work together to navigate the map, solve puzzles, and win! All are welcome.
[*]May 19th, 4:30pm PT/7:30pm ET, What's Next After Beast Academy?, designed for students finishing Beast Academy and ready for Prealgebra 1.
[*]May 20th, 4:00pm PT/7:00pm ET, Mathcamp 2025 Qualifying Quiz Part 1 Math Jam, Problems 1 to 4, join the Canada/USA Mathcamp staff for this exciting Math Jam, where they discuss solutions to Problems 1 to 4 of the 2025 Mathcamp Qualifying Quiz!
[*]May 21st, 4:00pm PT/7:00pm ET, Mathcamp 2025 Qualifying Quiz Part 2 Math Jam, Problems 5 and 6, Canada/USA Mathcamp staff will discuss solutions to Problems 5 and 6 of the 2025 Mathcamp Qualifying Quiz![/list]
Our full course list for upcoming classes is below:
All classes run 7:30pm-8:45pm ET/4:30pm - 5:45pm PT unless otherwise noted.

Introductory: Grades 5-10

Prealgebra 1 Self-Paced

Prealgebra 1
Tuesday, May 13 - Aug 26
Thursday, May 29 - Sep 11
Sunday, Jun 15 - Oct 12
Monday, Jun 30 - Oct 20
Wednesday, Jul 16 - Oct 29

Prealgebra 2 Self-Paced

Prealgebra 2
Wednesday, May 7 - Aug 20
Monday, Jun 2 - Sep 22
Sunday, Jun 29 - Oct 26
Friday, Jul 25 - Nov 21

Introduction to Algebra A Self-Paced

Introduction to Algebra A
Sunday, May 11 - Sep 14 (1:00 - 2:30 pm ET/10:00 - 11:30 am PT)
Wednesday, May 14 - Aug 27
Friday, May 30 - Sep 26
Monday, Jun 2 - Sep 22
Sunday, Jun 15 - Oct 12
Thursday, Jun 26 - Oct 9
Tuesday, Jul 15 - Oct 28

Introduction to Counting & Probability Self-Paced

Introduction to Counting & Probability
Thursday, May 15 - Jul 31
Sunday, Jun 1 - Aug 24
Thursday, Jun 12 - Aug 28
Wednesday, Jul 9 - Sep 24
Sunday, Jul 27 - Oct 19

Introduction to Number Theory
Friday, May 9 - Aug 1
Wednesday, May 21 - Aug 6
Monday, Jun 9 - Aug 25
Sunday, Jun 15 - Sep 14
Tuesday, Jul 15 - Sep 30

Introduction to Algebra B Self-Paced

Introduction to Algebra B
Tuesday, May 6 - Aug 19
Wednesday, Jun 4 - Sep 17
Sunday, Jun 22 - Oct 19
Friday, Jul 18 - Nov 14

Introduction to Geometry
Sunday, May 11 - Nov 9
Tuesday, May 20 - Oct 28
Monday, Jun 16 - Dec 8
Friday, Jun 20 - Jan 9
Sunday, Jun 29 - Jan 11
Monday, Jul 14 - Jan 19

Paradoxes and Infinity
Mon, Tue, Wed, & Thurs, Jul 14 - Jul 16 (meets every day of the week!)

Intermediate: Grades 8-12

Intermediate Algebra
Sunday, Jun 1 - Nov 23
Tuesday, Jun 10 - Nov 18
Wednesday, Jun 25 - Dec 10
Sunday, Jul 13 - Jan 18
Thursday, Jul 24 - Jan 22

Intermediate Counting & Probability
Wednesday, May 21 - Sep 17
Sunday, Jun 22 - Nov 2

Intermediate Number Theory
Sunday, Jun 1 - Aug 24
Wednesday, Jun 18 - Sep 3

Precalculus
Friday, May 16 - Oct 24
Sunday, Jun 1 - Nov 9
Monday, Jun 30 - Dec 8

Advanced: Grades 9-12

Olympiad Geometry
Tuesday, Jun 10 - Aug 26

Calculus
Tuesday, May 27 - Nov 11
Wednesday, Jun 25 - Dec 17

Group Theory
Thursday, Jun 12 - Sep 11

Contest Preparation: Grades 6-12

MATHCOUNTS/AMC 8 Basics
Friday, May 23 - Aug 15
Monday, Jun 2 - Aug 18
Thursday, Jun 12 - Aug 28
Sunday, Jun 22 - Sep 21
Tues & Thurs, Jul 8 - Aug 14 (meets twice a week!)

MATHCOUNTS/AMC 8 Advanced
Sunday, May 11 - Aug 10
Tuesday, May 27 - Aug 12
Wednesday, Jun 11 - Aug 27
Sunday, Jun 22 - Sep 21
Tues & Thurs, Jul 8 - Aug 14 (meets twice a week!)

AMC 10 Problem Series
Friday, May 9 - Aug 1
Sunday, Jun 1 - Aug 24
Thursday, Jun 12 - Aug 28
Tuesday, Jun 17 - Sep 2
Sunday, Jun 22 - Sep 21 (1:00 - 2:30 pm ET/10:00 - 11:30 am PT)
Monday, Jun 23 - Sep 15
Tues & Thurs, Jul 8 - Aug 14 (meets twice a week!)

AMC 10 Final Fives
Sunday, May 11 - Jun 8
Tuesday, May 27 - Jun 17
Monday, Jun 30 - Jul 21

AMC 12 Problem Series
Tuesday, May 27 - Aug 12
Thursday, Jun 12 - Aug 28
Sunday, Jun 22 - Sep 21
Wednesday, Aug 6 - Oct 22

AMC 12 Final Fives
Sunday, May 18 - Jun 15

AIME Problem Series A
Thursday, May 22 - Jul 31

AIME Problem Series B
Sunday, Jun 22 - Sep 21

F=ma Problem Series
Wednesday, Jun 11 - Aug 27

WOOT Programs
Visit the pages linked for full schedule details for each of these programs!


MathWOOT Level 1
MathWOOT Level 2
ChemWOOT
CodeWOOT
PhysicsWOOT

Programming

Introduction to Programming with Python
Thursday, May 22 - Aug 7
Sunday, Jun 15 - Sep 14 (1:00 - 2:30 pm ET/10:00 - 11:30 am PT)
Tuesday, Jun 17 - Sep 2
Monday, Jun 30 - Sep 22

Intermediate Programming with Python
Sunday, Jun 1 - Aug 24
Monday, Jun 30 - Sep 22

USACO Bronze Problem Series
Tuesday, May 13 - Jul 29
Sunday, Jun 22 - Sep 1

Physics

Introduction to Physics
Wednesday, May 21 - Aug 6
Sunday, Jun 15 - Sep 14
Monday, Jun 23 - Sep 15

Physics 1: Mechanics
Thursday, May 22 - Oct 30
Monday, Jun 23 - Dec 15

Relativity
Mon, Tue, Wed & Thurs, Jun 23 - Jun 26 (meets every day of the week!)
0 replies
jlacosta
May 1, 2025
0 replies
k i Adding contests to the Contest Collections
dcouchman   1
N Apr 5, 2023 by v_Enhance
Want to help AoPS remain a valuable Olympiad resource? Help us add contests to AoPS's Contest Collections.

Find instructions and a list of contests to add here: https://artofproblemsolving.com/community/c40244h1064480_contests_to_add
1 reply
dcouchman
Sep 9, 2019
v_Enhance
Apr 5, 2023
k i Zero tolerance
ZetaX   49
N May 4, 2019 by NoDealsHere
Source: Use your common sense! (enough is enough)
Some users don't want to learn, some other simply ignore advises.
But please follow the following guideline:


To make it short: ALWAYS USE YOUR COMMON SENSE IF POSTING!
If you don't have common sense, don't post.


More specifically:

For new threads:


a) Good, meaningful title:
The title has to say what the problem is about in best way possible.
If that title occured already, it's definitely bad. And contest names aren't good either.
That's in fact a requirement for being able to search old problems.

Examples:
Bad titles:
- "Hard"/"Medium"/"Easy" (if you find it so cool how hard/easy it is, tell it in the post and use a title that tells us the problem)
- "Number Theory" (hey guy, guess why this forum's named that way¿ and is it the only such problem on earth¿)
- "Fibonacci" (there are millions of Fibonacci problems out there, all posted and named the same...)
- "Chinese TST 2003" (does this say anything about the problem¿)
Good titles:
- "On divisors of a³+2b³+4c³-6abc"
- "Number of solutions to x²+y²=6z²"
- "Fibonacci numbers are never squares"


b) Use search function:
Before posting a "new" problem spend at least two, better five, minutes to look if this problem was posted before. If it was, don't repost it. If you have anything important to say on topic, post it in one of the older threads.
If the thread is locked cause of this, use search function.

Update (by Amir Hossein). The best way to search for two keywords in AoPS is to input
[code]+"first keyword" +"second keyword"[/code]
so that any post containing both strings "first word" and "second form".


c) Good problem statement:
Some recent really bad post was:
[quote]$lim_{n\to 1}^{+\infty}\frac{1}{n}-lnn$[/quote]
It contains no question and no answer.
If you do this, too, you are on the best way to get your thread deleted. Write everything clearly, define where your variables come from (and define the "natural" numbers if used). Additionally read your post at least twice before submitting. After you sent it, read it again and use the Edit-Button if necessary to correct errors.


For answers to already existing threads:


d) Of any interest and with content:
Don't post things that are more trivial than completely obvious. For example, if the question is to solve $x^{3}+y^{3}=z^{3}$, do not answer with "$x=y=z=0$ is a solution" only. Either you post any kind of proof or at least something unexpected (like "$x=1337, y=481, z=42$ is the smallest solution). Someone that does not see that $x=y=z=0$ is a solution of the above without your post is completely wrong here, this is an IMO-level forum.
Similar, posting "I have solved this problem" but not posting anything else is not welcome; it even looks that you just want to show off what a genius you are.

e) Well written and checked answers:
Like c) for new threads, check your solutions at least twice for mistakes. And after sending, read it again and use the Edit-Button if necessary to correct errors.



To repeat it: ALWAYS USE YOUR COMMON SENSE IF POSTING!


Everything definitely out of range of common sense will be locked or deleted (exept for new users having less than about 42 posts, they are newbies and need/get some time to learn).

The above rules will be applied from next monday (5. march of 2007).
Feel free to discuss on this here.
49 replies
ZetaX
Feb 27, 2007
NoDealsHere
May 4, 2019
Symmetry in Circumcircle Intersection
Mimii08   0
9 minutes ago
Hi! Here's another geometry problem I'm thinking about, and I would appreciate any help with a proof. Thanks in advance!

Let AD and BE be the altitudes of an acute triangle ABC, with D on BC and E on AC. The line DE intersects the circumcircle of triangle ABC again at two points M and N. Prove that CM = CN.

Thanks for your time and help!
0 replies
Mimii08
9 minutes ago
0 replies
Polynomial of Degree n
Brut3Forc3   20
N 23 minutes ago by Ilikeminecraft
Source: 1975 USAMO Problem 3
If $ P(x)$ denotes a polynomial of degree $ n$ such that $ P(k)=\frac{k}{k+1}$ for $ k=0,1,2,\ldots,n$, determine $ P(n+1)$.
20 replies
Brut3Forc3
Mar 15, 2010
Ilikeminecraft
23 minutes ago
Really fun geometry problem
Sadigly   5
N 39 minutes ago by GingerMan
Source: Azerbaijan Senior MO 2025 P6
In the acute triangle $ABC$ with $AB<AC$, the foot of altitudes from $A,B,C$ to the sides $BC,CA,AB$ are $D,E,F$, respectively. $H$ is the orthocenter. $M$ is the midpoint of segment $BC$. Lines $MH$ and $EF$ intersect at $K$. Let the tangents drawn to circumcircle $(ABC)$ from $B$ and $C$ intersect at $T$. Prove that $T;D;K$ are colinear
5 replies
Sadigly
Today at 4:29 PM
GingerMan
39 minutes ago
Polynomials and powers
rmtf1111   27
N 44 minutes ago by bjump
Source: RMM 2018 Day 1 Problem 2
Determine whether there exist non-constant polynomials $P(x)$ and $Q(x)$ with real coefficients satisfying
$$P(x)^{10}+P(x)^9 = Q(x)^{21}+Q(x)^{20}.$$
27 replies
rmtf1111
Feb 24, 2018
bjump
44 minutes ago
Equilateral triangle formed by circle and Fermat point
Mimii08   0
an hour ago
Source: Heard from a friend
Hi! I found this interesting geometry problem and I would really appreciate help with the proof.

Let ABC be an acute triangle, and let T be the Fermat (Torricelli) point of triangle ABC. Let A1, B1, and C1 be the feet of the perpendiculars from T to the sides BC, AC, and AB, respectively. Let ω be the circle passing through points A1, B1, and C1. Let A2, B2, and C2 be the second points where ω intersects the sides BC, AC, and AB, respectively (different from A1, B1, C1).

Prove that triangle A2B2C2 is equilateral.

0 replies
Mimii08
an hour ago
0 replies
Problem 3 IMO 2005 (Day 1)
Valentin Vornicu   121
N an hour ago by Rayvhs
Let $x,y,z$ be three positive reals such that $xyz\geq 1$. Prove that
\[ \frac { x^5-x^2 }{x^5+y^2+z^2} + \frac {y^5-y^2}{x^2+y^5+z^2} + \frac {z^5-z^2}{x^2+y^2+z^5} \geq 0 . \]
Hojoo Lee, Korea
121 replies
Valentin Vornicu
Jul 13, 2005
Rayvhs
an hour ago
geo problem saved from graveyard
CrazyInMath   1
N 2 hours ago by Curious_Droid
Source: 3rd KYAC Math-A P5
Given triangle $ABC$ and orthocenter $H$. The foot from $H$ to $BC, CA, AB$ is $D, E, F$ respectively. A point $L$ satisfies that $\odot(LBA)$ and $\odot(LCA)$ are both tangent to $BC$. A circle passing through $B, E$ and tangent to $\odot(BHC)$ intesects $BC$ at another point $P$. $X$ is an arbitrary point on $\odot(PDE)$, and $Y$ is the second intesection point of $\odot(BXE)$ and $\odot(CXD)$.
Prove that $H, Y, L, C$ are concyclic.

Proposed by CrazyInMath.
1 reply
CrazyInMath
Feb 8, 2025
Curious_Droid
2 hours ago
From a well-known prob
m4thbl3nd3r   3
N 2 hours ago by aaravdodhia
Find all primes $p$ so that $$\frac{7^{p-1}-1}{p}$$can be a perfect square
3 replies
m4thbl3nd3r
Oct 10, 2024
aaravdodhia
2 hours ago
weird conditions in geo
Davdav1232   1
N 2 hours ago by NO_SQUARES
Source: Israel TST 7 2025 p1
Let \( \triangle ABC \) be an isosceles triangle with \( AB = AC \). Let \( D \) be a point on \( AC \). Let \( L \) be a point inside the triangle such that \( \angle CLD = 90^\circ \) and
\[
CL \cdot BD = BL \cdot CD.
\]Prove that the circumcenter of triangle \( \triangle BDL \) lies on line \( AB \).
1 reply
Davdav1232
3 hours ago
NO_SQUARES
2 hours ago
Functional equation on R
rope0811   15
N 2 hours ago by ezpotd
Source: IMO ShortList 2003, algebra problem 2
Find all nondecreasing functions $f: \mathbb{R}\rightarrow\mathbb{R}$ such that
(i) $f(0) = 0, f(1) = 1;$
(ii) $f(a) + f(b) = f(a)f(b) + f(a + b - ab)$ for all real numbers $a, b$ such that $a < 1 < b$.

Proposed by A. Di Pisquale & D. Matthews, Australia
15 replies
rope0811
Sep 30, 2004
ezpotd
2 hours ago
all functions satisfying f(x+yf(x))+y = xy + f(x+y)
falantrng   34
N 3 hours ago by LenaEnjoyer
Source: Balkan MO 2025 P3
Find all functions $f\colon \mathbb{R} \rightarrow \mathbb{R}$ such that for all $x,y \in \mathbb{R}$,
\[f(x+yf(x))+y = xy + f(x+y).\]
Proposed by Giannis Galamatis, Greece
34 replies
falantrng
Apr 27, 2025
LenaEnjoyer
3 hours ago
Miklos Schweitzer 1971_7
ehsan2004   1
N 3 hours ago by pi_quadrat_sechstel
Let $ n \geq 2$ be an integer, let $ S$ be a set of $ n$ elements, and let $ A_i , \; 1\leq i \leq m$, be distinct subsets of $ S$ of size at least $ 2$ such that \[ A_i \cap A_j \not= \emptyset, A_i \cap A_k \not= \emptyset, A_j \cap A_k \not= \emptyset, \;\textrm{imply}\ \;A_i \cap A_j \cap A_k \not= \emptyset \ .\] Show that $ m \leq 2^{n-1}-1$.

P. Erdos
1 reply
ehsan2004
Oct 29, 2008
pi_quadrat_sechstel
3 hours ago
Functional equation with a twist (it's number theory)
Davdav1232   0
3 hours ago
Source: Israel TST 8 2025 p2
Prove that for all primes \( p \) such that \( p \equiv 3 \pmod{4} \) or \( p \equiv 5 \pmod{8} \), there exist integers
\[
1 \leq a_1 < a_2 < \cdots < a_{(p-1)/2} < p
\]such that
\[
\prod_{\substack{1 \leq i < j \leq (p-1)/2}} (a_i + a_j)^2 \equiv 1 \pmod{p}.
\]
0 replies
Davdav1232
3 hours ago
0 replies
Grid combi with T-tetrominos
Davdav1232   0
3 hours ago
Source: Israel TST 8 2025 p1
Let \( f(N) \) denote the maximum number of \( T \)-tetrominoes that can be placed on an \( N \times N \) board such that each \( T \)-tetromino covers at least one cell that is not covered by any other \( T \)-tetromino.

Find the smallest real number \( c \) such that
\[
f(N) \leq cN^2
\]for all positive integers \( N \).
0 replies
Davdav1232
3 hours ago
0 replies
IMO 2018 Problem 5
orthocentre   80
N Yesterday at 10:10 PM by OronSH
Source: IMO 2018
Let $a_1$, $a_2$, $\ldots$ be an infinite sequence of positive integers. Suppose that there is an integer $N > 1$ such that, for each $n \geq N$, the number
$$\frac{a_1}{a_2} + \frac{a_2}{a_3} + \cdots + \frac{a_{n-1}}{a_n} + \frac{a_n}{a_1}$$is an integer. Prove that there is a positive integer $M$ such that $a_m = a_{m+1}$ for all $m \geq M$.

Proposed by Bayarmagnai Gombodorj, Mongolia
80 replies
orthocentre
Jul 10, 2018
OronSH
Yesterday at 10:10 PM
IMO 2018 Problem 5
G H J
Source: IMO 2018
The post below has been deleted. Click to close.
This post has been deleted. Click here to see post.
orthocentre
72 posts
#1 • 27 Y
Y by Durjoy1729, Moaaz, me9hanics, MazeaLarius, Amir Hossein, Davi-8191, naw.ngs, Medjl, adityaguharoy, anantmudgal09, pavel kozlov, khan.academy, centslordm, megarnie, Sprites, jasperE3, myh2910, mathmax12, NO_SQUARES, Adventure10, Mango247, aidan0626, torch, buddyram, deplasmanyollari, Supercali, MS_asdfgzxcvb
Let $a_1$, $a_2$, $\ldots$ be an infinite sequence of positive integers. Suppose that there is an integer $N > 1$ such that, for each $n \geq N$, the number
$$\frac{a_1}{a_2} + \frac{a_2}{a_3} + \cdots + \frac{a_{n-1}}{a_n} + \frac{a_n}{a_1}$$is an integer. Prove that there is a positive integer $M$ such that $a_m = a_{m+1}$ for all $m \geq M$.

Proposed by Bayarmagnai Gombodorj, Mongolia
This post has been edited 3 times. Last edited by djmathman, Jun 16, 2020, 4:03 AM
Reason: problem author
Z K Y
The post below has been deleted. Click to close.
This post has been deleted. Click here to see post.
Itama
78 posts
#2 • 4 Y
Y by Kgxtixigct, Adventure10, Mango247, buddyram
I can sence the beauty of the IMO 2018 problems... :coolspeak:

Exelent Job!!! problem selection committee & team leaders!!

I would like to know whose problem is this?
This post has been edited 2 times. Last edited by Itama, Jul 10, 2018, 12:10 PM
Z K Y
The post below has been deleted. Click to close.
This post has been deleted. Click here to see post.
mathocean97
606 posts
#3 • 16 Y
Y by e_plus_pi, pieater314159, UK2019Project, Pluto1708, Siddharth03, myh2910, IcCircle, centslordm, megarnie, Nuterrow, mathmax12, Adventure10, Mango247, Zhaom, aidan0626, buddyram
Main claim: If $\frac{a}{x} + \frac{x}{b} - \frac{a}{b}$ is an integer, then $\gcd(a, b) | x.$

Proof: Compute the expression. It becomes $\frac{ab+x^2-ax}{bx}.$ Say a prime $p$ satisfies $p|a, p|b.$ Then $p|x^2$ so $p|x$. Divide it out and continue.

Now, I will show that as the sequence goes along, that both the numerator and denominator of the reduced fraction $\frac{a_n}{a_1}$ will decrease as $n$ increases past $n \ge N.$

Now, assume that $\frac{a}{x} + \frac{x}{b} - \frac{a}{b}$ is an integer and $\gcd(a, b) = 1.$ Then $x | ab$ so $x = a'b'$ where $a' | a, b' | b$. Now take $a = a_n, b = a_1, x = a_{n+1}.$ Then $\frac{a_{n+1}}{a_1} = \frac{x}{b} = \frac{a'}{(b/b')}.$ So both the numerator and denominator decreased! Therefore, the sequence $\frac{a_n}{a_1}$ will eventually converge, as desired.
Z K Y
The post below has been deleted. Click to close.
This post has been deleted. Click here to see post.
v_Enhance
6877 posts
#5 • 25 Y
Y by samoha, Davi-8191, brokendiamond, Mathlover1292, qweDota, rashah76, Abidabi, v4913, Sunnybest, centslordm, megarnie, HamstPan38825, myh2910, Assassino9931, mathmax12, Danielzh, Adventure10, Mango247, buddyram, Sneakyturtle, sky.mty, DEKT, Ali_Vafa, Funcshun840, MS_asdfgzxcvb
The condition implies that the difference $S(n) = \frac{a_{n+1}}{a_1} - \frac{a_n}{a_1} + \frac{a_n}{a_{n+1}}$ is an integer for all $n > N$. We proceed by $p$-adic valuation only henceforth.

Claim: If $p \nmid a_1$, then $\nu_p(a_{n+1}) \le \nu_p(a_n)$ for $n \ge N$.

Proof. The first two terms of $S(n)$ have nonnegative $\nu_p$, so we need $\nu_p(\frac{a_n}{a_{n+1}}) \ge 0$. $\blacksquare$

Claim: If $p \mid a_1$, then $\nu_p(a_n)$ is eventually constant.

Proof. By hypothesis $\nu_p(a_1) > 0$. We consider two cases.
  • First assume $\nu_p(a_k) \ge \nu_p(a_1)$ for some $k > N$. We claim that for any $n \ge k$ we have: \[ \nu_p(a_1) \le \nu_p(a_{n+1}) \le \nu_p(a_n). \]This is just by induction on $n$; from $\nu(\frac{a_n}{a_1}) \ge 0$, we have \[ \nu_p\left( \frac{a_{n+1}}{a_1} + \frac{a_n}{a_{n+1}} \right) \ge 0 \]which implies the displayed inequality (since otherwise exactly one term of $S(n)$ has nonnegative $\nu_p$). Thus once we reach this case, $\nu_p(a_n)$ is monotic but bounded below by $\nu_p(a_1)$, and so it is eventually constant.
  • Now assume $\nu_p(a_k) < \nu_p(a_1)$ for every $k > N$. Take any $n > N$ then. We have $\nu_p\left(\frac{a_{n+1}}{a_1}\right) < 0$, and also $\nu_p\left(\frac{a_n}{a_1}\right) < 0$, so among the three terms of $S(n)$, two must have equal $p$-adic valuation. We consider all three possibilities: \begin{align*} 			\nu_p\left(\frac{a_{n+1}}{a_1}\right) = \nu_p\left(\frac{a_n}{a_1}\right) 			&\implies \boxed{\nu_p(a_{n+1}) = \nu_p(a_{n})} \\ 			\nu_p\left(\frac{a_{n+1}}{a_1}\right) = \nu_p\left(\frac{a_n}{a_{n+1}}\right) 			&\implies \boxed{\nu_p(a_{n+1}) = \frac{\nu_p(a_n) + \nu_p(a_1)}{2}} \\ 			\nu_p\left(\frac{a_{n}}{a_1}\right) = \nu_p\left(\frac{a_n}{a_{n+1}}\right) 			&\implies \nu_p(a_{n+1}) = \nu_p(a_1),\text{ but this is impossible}. 		\end{align*}Thus, $\nu_p(a_{n+1}) \ge \nu_p(a_n)$ and $\nu_p(a_n)$ is bounded above by $\nu_p(a_1)$. So in this case we must also stabilize. \qedhere
$\blacksquare$

Since the latter claim is applied only to finitely many primes, after some time $\nu_p(a_n)$ is fixed for all $p \mid a_1$. Afterwards, the sequence satisfies $a_{n+1} \mid a_n$ for each $n$, and thus must be eventually constant.

Remark: This solution is almost completely $p$-adic, in the sense that I think a similar result if one replaces $a_n \in {\mathbb Z}$ by $a_n \in {\mathbb Z}_p$ for any particular prime $p$. In other words, the primes almost do not talk to each other.

There is one caveat: if $x_n$ is an integer sequence such that $\nu_p(x_n)$ is eventually constant for each prime then $x_n$ may not be constant. For example, take $x_n$ to be the $n$th prime! That's why in the first claim (applied to co-finitely many of the primes), we need the stronger non-decreasing condition, rather than just eventually constant.
This post has been edited 6 times. Last edited by v_Enhance, Aug 21, 2018, 2:47 AM
Reason: colorize
Z K Y
The post below has been deleted. Click to close.
This post has been deleted. Click here to see post.
knm2608
468 posts
#6 • 3 Y
Y by adityaguharoy, Adventure10, buddyram
Written a bit hastily, so let me know if there is any flaw.

We have that
$$\frac{a_n}{a_{n+1}}+\frac{a_{n+1}-a_n}{a_1} \in \mathbb{Z} \;\;\; \forall n \geq N \;\;\;\;\;\; (1)$$Now let $m>n$ be an integer and
$$A=\frac{a_1}{a_2} + \frac{a_2}{a_3} + \ldots + \frac{a_{n-1}}{a_n}, \;\;\;\; B=\frac{a_n}{a_{n+1}}+\ldots +\frac{a_{m-1}}{a_m}$$Then
$$B+\frac{a_m}{a_1}-\frac{a_n}{a_1}\in \mathbb{Z} \;\;\;\;(2)$$Taking $(1)$ from $n$ to $m$ combined with $(2)$ gives
$$\frac{a_m}{a_1} \in \mathbb{Z} \; \Rightarrow \frac{a_m}{a_{m+1}}\in \mathbb{Z} \;\;\; \forall m > n$$So $a_{n+1}=ka_1,k\in \mathbb{Z}$. But
$$\frac{a_n}{a_1}\left(1-\frac{1}{k} \right)  \in \mathbb{Z}$$So either $k=1$ which implies $a_{n+1}=a_1 \Rightarrow a_m=a_{m+1} \;\; \forall m\geq n+1$ or $a_1|a_n \Rightarrow a_{n+1}|a_n \;\; \forall n$. In either case we're done.

Edit: It is wrong. Thanks below.
This post has been edited 1 time. Last edited by knm2608, Jul 11, 2018, 12:25 PM
Z K Y
The post below has been deleted. Click to close.
This post has been deleted. Click here to see post.
Kurt Gödel
615 posts
#7 • 4 Y
Y by Doraeminion, Adventure10, Mango247, buddyram
knm2608 wrote:
Taking $(1)$ from $n$ to $m$ combined with $(2)$ gives $\frac{a_m}{a_1} \in \mathbb{Z}$

How? I think (2) is just a repeated application of (1), so it seems like the most you will get out of this is a tautology.

Great problem though!
Z K Y
The post below has been deleted. Click to close.
This post has been deleted. Click here to see post.
WizardMath
2487 posts
#9 • 5 Y
Y by rashah76, Adventure10, Mango247, bhan2025, buddyram
Since the sequence is an integer sequence, the differences of consecutive terms must also be integers. So we have $$\frac{a_{n+1}^2-a_n a_{n+1} +  a_n a_1}{a_1 a_{n+1}} \in \mathbb{Z}$$Subtracting 1, we have $a_1 a_{n+1} | (a_{n+1} - a_1) (a_{n+1} - a_n)$. Now consider any prime $p$ that divides $a_1, a_{n}$. Then we have, by dividing both sides by $p$, that $p| a_{n+1}$. Now divide all of $a_1, a_n, a_{n+1}$ by $p$, and then notice that we can still argue further like this. So $\min ( \nu_p (a_1), \nu_p (a_{n}) ) \le \nu_p(a_{n+1}) $. Since we have $a_1 a_{n+1} | (a_{n+1} - a_1) (a_{n+1} - a_n)$, we see that $a_{n+1} | a_1 a_n$.
Note that if $\gcd (a_1, a_{n+1}) = \alpha_{n+1},$ then $a_{n+1} / \alpha_{n+1}$ is an integer, and it divides $a_n$, so the resulting conjugate factor is $k$ (say). Then $\frac{a_{n+1}}{a_1} = \frac{a_n/k}{a_{1}/\alpha_{n+1}}$, so the moduli of numerator and denominator are non-increasing. This leads us to $a_m = a_{m+1}$ from some $m$ onwards, since no positive integer sequence can strictly decrease forever.
This post has been edited 2 times. Last edited by WizardMath, Jul 10, 2018, 2:54 PM
Z K Y
The post below has been deleted. Click to close.
This post has been deleted. Click here to see post.
Loppukilpailija
155 posts
#11 • 4 Y
Y by amar_04, Adventure10, Mango247, buddyram
Here's a solution I wrote during the competition. Quite similar to the one written by v_Enhance, but the execution is maybe a little bit bloodier.


Solution:

Proof by contradiction. For the rest of the solution we assume that the sequence $a_n$ isn't constant starting from any point. In the end we will achieve a contradiction, thus proving the statement.

The difference

$$ \frac{a_{n+1}}{a_1} + \frac{a_n}{a_{n+1}} - \frac{a_n}{a_1} = \frac{a_{n+1}^2 - a_{n+1}a_n + a_na_1}{a_1a_{n+1}} $$
is an integer for all $n \ge N$.

Lemma 1.

There is a prime $p$ such that $v_p(a_{n+1}) \neq v_p(a_n)$ for infinitely many $n \ge N$.

Proof.

The integer-ness condition above implies that $a_{n+1} \mid a_na_1$. This means that for all $p \mid a_n$, $p$ prime and $n \ge N$, we have either $p \mid a_N$ or $p \mid a_1$ (assume not, and take the smallest $i > N$ for which $p \mid a_i$). Thus, there are only finitely many $p$ such that $p \mid a_n$ for some $n \ge N$. Now, for the statement of the lemma, assume not: now, if we take any prime $p$ which doesn't divide any of the numbers $a_1, a_N, a_{N+1}, a_{N+2}, \ldots $, then we of course have $v_p(a_{n+1}) = v_p(a_n)$ for all $n \ge N$. For the rest of the primes, which we have finitely many, we must have $v_p(a_{n+1}) = v_p(a_n)$ for all large enough $n$. But this means that the sequence $a_n$ is constant, which is a contradiction. Thus, the statement of the lemma is true.

Now, take any prime $p$ satisfying the condition of lemma 1. Define $v_p(a_n) = b_n$ for all $n$. We aren't that interested in the cases where $b_{n+1} = b_n$, but luckily we have infinitely many interesting cases.

Lemma 2. If $b_{n+1} > b_n$, we have $b_{n+1} = b_1$.

Proof: we divide into two cases, one where $b_{n+1} > b_1$, and one where $b_{n+1} < b_1$. We prove that both of these cases lead into a contradiction, which proves the lemma.

If $b_{n+1} > b_1$, then $v_p(a_{n+1}^2 - a_na_{n+1} + a_na_1) = v_p(a_na_1) = b_n + b_1 < b_{n+1} + b_1 = v_p(a_{n+1}a_1)$, contradicting the fact that $a_{n+1}a_1$ divides $a_{n+1}^2 - a_na_{n+1} + a_na_1$.
In a similar manner we see that if $b_{n+1} < b_1$, then the $v_p$ of the denominator is $b_n + b_{n+1}$, which is less than $b_{n+1} + b_1$, since $b_1 > b_{n+1} > b_n$. Thus, we must have $b_{n+1} = b_1$.

Lemma 3.
If $b_{n+1} < b_n$, then we have $b_1 \ge b_{n+1}$.

Proof. Again, a proof by contradiction. Now, the $v_p$ of the denominator is $2b_{n+1}$, which is less than $b_1 + b_{n+1}$.

Now, we have our setup ready. We only need the following crucial lemma, and then we are basically done:

Lemma 4. There exists an $n \ge N$ such that $b_n = b_1$.

Proof. Surprise, a proof by contradiction. Now, if $b_{n+1} > b_n$ for some $n$, then $b_{n+1} = b_1$ due to lemma 2. This can't happen, so $b_{n+1} \le b_n$ for all $n$. So $b_n$ is a decreasing sequence, which is a contradiction with the fact that $b_{n+1} \neq b_n$ for infinitely many $n$, and that $b_n \ge 0$ for all $n$.

Now, take such $n$ that $b_n = b_1$. If $b_{n+1} = b_n$, just increment $n$ by $1$ as long as we have $b_{n+1} \neq b_n = b_1$. This gives a contradiction:

if $b_{n+1} > b_n$, then due to lemma 2 we have $b_1 = b_{n+1} > b_n = b_1$, a contradiction.

if $b_{n+1} < b_n$, then due to lemma 3 we have $b_1 \le b_{n+1} < b_n = b_1$, a contradiction.

Thus, the original assumption of the statement being false gave a contradiction. The sequence $a_m$ is therefore constant starting from some point.

Motivation:


The divisibility condition given is very nice for investigating the $p$-adic valuations, as there are no constant terms. Then, you just bash the cases a little bit to use the well-known lemma $v_p(a \pm b) = \min(v_p(a), v_p(b))$ for $v_p(a) \neq v_p(b)$. I was hoping for a direct contradiction in either one of the cases $b_{n+1} > b_n$ or $b_{n+1} < b_n$, but I wasn't able do get this, so I just picked up all the nice information I got from the case-work there. When you really look at lemmas 2 and 3, it's not too difficult to come up with the rest of the solution. Lemma 1 is only needed to describe the prime we pick for our "$v_p$-bash".
This post has been edited 2 times. Last edited by Loppukilpailija, Jul 17, 2018, 4:05 PM
Reason: Add motivation.
Z K Y
The post below has been deleted. Click to close.
This post has been deleted. Click here to see post.
62861
3564 posts
#12 • 12 Y
Y by Loppukilpailija, nmd27082001, pavel kozlov, Reef334, XbenX, edfearay123, centslordm, leozitz, Wizard0001, Adventure10, Mango247, buddyram
The difference $\frac{a_{n+1}}{a_1} - \frac{a_n}{a_1} + \frac{a_n}{a_{n+1}}$ is also an integer for $n \ge N$.
Hence
\[a_1 a_{n+1} \mid a_{n+1}^2 - a_n a_{n+1} + a_1 a_n \equiv (a_{n+1} - a_1)(a_{n+1} - a_n).\]
Lemma. If $a$, $x$, $y$ are positive integers with $ay \mid (y - a)(y - x)$ and $p$ is a prime, then
\[\min\big(\nu_p(a), \nu_p(x)\big) \le \nu_p(y) \le \max\big(\nu_p(a), \nu_p(x)\big).\]This can be easily proved by examining the possibilities $\nu_p(y) < \min\big(\nu_p(a), \nu_p(x)\big)$ and $\nu_p(y) > \max\big(\nu_p(a), \nu_p(x)\big)$; both lead to $\nu_p$ contradictions in the divisibility.

Corollary 1. $\gcd(a_1, a_n) \mid a_{n+1} \mid \mathrm{lcm}(a_1, a_n)$ for $n \ge N$.

Corollary 2. $\gcd(a_1, a_n) \mid \gcd(a_1, a_{n+1})$ and $\mathrm{lcm}(a_1, a_{n+1}) \mid \mathrm{lcm}(a_1, a_n)$ for $n \ge N$.

Hence $\{\gcd(a_1, a_n)\}_{n \ge N}$ and $\{\mathrm{lcm}(a_1, a_n)\}_{n \ge N}$ are increasing and decreasing sequences (respectively) bounded by $a_1$. Thus they are eventually constant; say $\gcd(a_1, a_m) = u$ and $\mathrm{lcm}(a_1, a_m) = v$ for $m \ge M$. Then
\[a_m = \frac{\gcd(a_1, a_m) \cdot \mathrm{lcm}(a_1, a_m)}{a_1} = \frac{uv}{a_1}\]for all $m \ge M$, as desired.
This post has been edited 1 time. Last edited by 62861, Jul 10, 2018, 5:45 PM
Z K Y
The post below has been deleted. Click to close.
This post has been deleted. Click here to see post.
yugrey
2326 posts
#13 • 3 Y
Y by Adventure10, Mango247, buddyram
v_Enhance wrote:
The condition implies that the difference \[ \frac{a_{n+1}}{a_1} - \frac{a_n}{a_1} + \frac{a_n}{a_{n+1}} \]is an integer for all $n > N$.

We proceed by $p$-adic valuation only henceforth.

Claim: If $p \nmid a_1$, then $\nu_p(a_{n+1}) \le \nu_p(a_n)$ for $n \ge N$.

Proof. The third term in the difference must have nonnegative $\nu_p$. $\blacksquare$



Claim: If $p \mid a_1$, then $\nu_p(a_n)$ is eventually constant.

Proof. By hypothesis $\nu_p(a_1) > 0$. We consider two cases,
  • First assume $\nu_p(a_k) \ge \nu_p(a_1)$ for some $k > N$. We claim that for any $n \ge k$ (by induction) we have: \[ \nu_p(a_1) \le \nu_p(a_{n+1}) \le \nu_p(a_n). \]Note in this case that $\nu(\frac{a_n}{a_1}) \ge 0$, so we must have \[ \nu_p\left( \frac{a_{n+1}}{a_1} + \frac{a_n}{a_{n+1}} \right) \ge 0 \]which implies the displayed inequality (since otherwise exactly one of the summands has nonnegative $\nu_p$). Thus once we reach this case, $\nu_p(a_n)$ is monotic but bounded below by $\nu_p(a_1)$, and so it is eventually constant.
  • Now assume $\nu_p(a_k) < \nu_p(a_1)$ for every $k > N$. Take any $n > N$ then. We have $\nu_p\left(\frac{a_{n+1}}{a_1}\right) < 0$, and also $\nu_p\left(\frac{a_n}{a_1}\right) < 0$, so among the three summands two must have equal $p$-adic valuation. We consider all three possibilities: \begin{align*} 			\nu_p\left(\frac{a_{n+1}}{a_1}\right) = \nu_p\left(\frac{a_n}{a_1}\right) 			&\implies \boxed{\nu_p(a_{n+1}) = \nu_p(a_{n})} \\ 			\nu_p\left(\frac{a_{n+1}}{a_1}\right) = \nu_p\left(\frac{a_n}{a_{n+1}}\right) 			&\implies \boxed{\nu_p(a_{n+1}) = \frac{\nu_p(a_n) + \nu_p(a_1)}{2}} \\ 			\nu_p\left(\frac{a_{n}}{a_1}\right) = \nu_p\left(\frac{a_n}{a_{n+1}}\right) 			&\implies \nu_p(a_{n+1}) = \nu_p(a_1),\text{ but this is impossible}. 		\end{align*}Thus, eventually $\nu_p(a_{n+1}) \ge \nu_p(a_n)$ in either possible situation, but $\nu_p(a_n)$ is bounded above by $\nu_p(a_1) - 1$, so in this case we must also stabilize. \qedhere
$\blacksquare$

Since the latter claim is applied only to finitely many primes, after some time $\nu_p(a_n)$ is fixed for all $p \nmid a_n$. Afterwards, the sequence satisfies $a_{n+1} \mid a_n$ for each $n$, and thus must be eventually constant.

Remark: This solution is almost completely $p$-adic, in the sense that I think a similar result if one replaces $a_n \in {\mathbb Z}$ by $a_n \in {\mathbb Z}_p$ for any particular prime $p$. In other words, the primes almost do not talk to each other.

There is one caveat: if $x_n$ is an integer sequence such that $\nu_p(x_n)$ is eventually constant for each prime then $x_n$ may not be constant. For example, take $x_n$ to be the $n$th prime! That's why in the first claim (applied to co-finitely many of the primes), we need the stronger non-decreasing condition, rather than just eventually constant.

This is basically my solution. This "caveat" at the end is easy to get around if you notice that since $\frac{a_{n+1}^2-a_na_{n+1}+a_na_1}{a_1a_{n+1}}$ is an integer, $a_{n+1} \mid a_na_1$, so eventually $\frac{a_n}{(a_n,a_1)}$ converges to a constant. Then you basically do this analysis only on primes that divide $a_1$, of which there are finitely many. I think this is the more intuitive way to deal with it. :)
Z K Y
The post below has been deleted. Click to close.
This post has been deleted. Click here to see post.
suli
1498 posts
#14 • 3 Y
Y by Adventure10, Mango247, buddyram
For any prime $p$, $v_p (a_{n+1})$ is between $v_p (a_n)$ and $v_p (a_1)$. That's it. Was expecting more from an IMO P5 :(

Solution
Z K Y
The post below has been deleted. Click to close.
This post has been deleted. Click here to see post.
JuanOrtiz
366 posts
#15 • 2 Y
Y by Adventure10, Mango247
Let $x=a_1$. Throughout the problem, $n$ will denote an arbitrary number $\ge N$. Notice
\[ \frac{a_{n}}{a_{n+1}} + \frac{a_{n+1}-a_n}{x} = \left( \frac{a_1}{a_2}+\cdots + \frac{a_n}{a_{n+1}} + \frac{a_{n+1}}{x} \right) -\left( \frac{a_1}{a_2}+\cdots + \frac{a_n}{x}  \right)  \]is an integer. Thus $a_{n+1}=\frac{a_nx}{\ell_n}$ for some positive integer $\ell_n$ such that $x| \ell_n+a_{n+1}-a_n$. Thus, for any prime $p\nmid x$, we have $v_p(a_n)$ is decreasing, and so eventually becomes constant. There's finitely many primes that divide $a_N$, and so there's some $M\ge N$ such that for any $p\nmid x$, the quantity $v_p(a_n)$ is fixed after $n\ge M$, and the $\ell_n$'s are only divisible by primes in $P:=\{ p | x\}$, i.e. primes that divide $x$. From now on, $n$ will denote an arbitrary number $\ge M$.

Let $p\in P$ and set $b:= v_p(x)$, $b_n := v_p(a_n)$. Assume for some $n$, $b_{n+1}> b_n$, so that $b > v_p(\ell_n)$. Notice $v_p(a_{n+1}-a_n) = b_n$ but $b\le v_p(\ell_n+a_{n+1}-a_n)$, and so $b_n=v_p(\ell_n) < b$. Thus $b_{n+1}=b$ and so $p^b | \ell_{n+1} + a_{n+2}$. But $v_p(\ell_{n+1}) + v_p(a_{n+2}) = 2b$ and so both must be $b$, thus $b_{n+2}=b$ and thus the sequence $\{b_i\}$ is eventually constant. Otherwise, the sequence is never increasing, but it can never be negative, so it's also eventually constant.

Since $P$ is finite, and all these sequences are constant, we get the $P$-part of the $\{a_n\}$ sequence is eventually constant. But the first paragraph proves the non-$P$ part is also eventually constant. Thus, the sequence itself is eventually constant as desired.
Z K Y
The post below has been deleted. Click to close.
This post has been deleted. Click here to see post.
Taha1381
816 posts
#16 • 2 Y
Y by Adventure10, buddyram
The proof is quite straight for a problem $5$.

Lemma:There are finite primes dividing one or more elements of the sequence.

Proof:Assume for country then there exists a prime $p$ so that $p$ divides $a_{t+1}$ for some large $t$ but not the previous elements of the subset.Since
$\frac{a_{t+1}}{a_1} - \frac{a_t}{a_1} + \frac{a_t}{a_{t+1}}$ has to be an integer this is impossible.

Let $\mathbb{P}$ denote the set of primes dividing at least one element of the sequence and take $p \in \mathbb{P}$.We divide the problem into two cases.

First case:$v_p(a_n) <v_p(a_1)$ holds for all integer $n \ge N$:Since $\frac{a_{n+1}}{a_1} - \frac{a_n}{a_1} + \frac{a_n}{a_{n+1}}$ has to be an integer using nothing more than $p$-adic calculations one can show $v_p(a_N) \le v_p(a_{N+!} \le \dots$ because the sequence of $v_p(a_i)$ has an upper bound it has to be constant after somewhere.

Second case :There exist $\ell \ge N$ so that $v_p(a_{\ell }) \ge v_p(a_1)$.By induction and some $p$-adic calculations one can prove $v_p(a_{\ell }) \ge v_p(a_{\ell +1}) \ge \dots $ and all this values are bigger than or equal to $v_p(a_1)$ so by FMID it has to be constant after somewhere.

So there exist an $M$ so that powers of all primes in $\mathbb{P}$ is constant in $a_M,a_{M+1},\dots $ since they are the only primes dividing one or more element of the sequence hence all these numbers have to be equal.

Is this solution true?If so then why is this a problem $5$(I came up with it within 30 minutes in the mock imo we had in Iran while I spend some hours or days to solve some other problems 5's).

Remark:I noticed this is a way to deal with the issue evan remarked in his answer to show the wrong appropach..
This post has been edited 3 times. Last edited by Taha1381, Jul 11, 2018, 1:04 PM
Z K Y
The post below has been deleted. Click to close.
This post has been deleted. Click here to see post.
reality95
12 posts
#17 • 4 Y
Y by rmtf1111, Adventure10, Mango247, buddyram
More or less the same idea as above.
Suppose that $n \ge N$ and $a_1 = c$ then we get that $$ca_{n + 1} | a_{n+1}^2 + a_nc - a_na_{n+1}$$
We must have for every prime $p$ $v_p(c) + v_p(a_{n + 1}) \le v_p(a_{n+1}^2 + a_nc - a_na_{n+1})$.

$\textbf{Lemma 1.}$ $v_p(a_{n + 1}) \le \max(v_p(c),v_p(a_n))$

Proof: Suppose the contrary, then $v_p(c) + v_p(a_n) < v_p(a_n) + v_p(a_{n + 1}) < 2v_p(a_{n + 1})$ so $v_p(c) + v_p(a_{n + 1}) \le v_p(c) + v_p(a_n)$ or $v_p(a_{n + 1}) \le v_p(a_n)$, contradiction.

$\textbf{Lemma 2.}$ If $v_p(a_{n + 1}) < v_p(a_n)$ then $v_p(a_{n + 1}) \ge v_p(c)$

Proof: Suppose $v_p(a_{n + 1}) < v_p(c) \le v_p(a_n)$ then $2v_p(a_{n + 1}) < v_p(a_{n + 1}) + v_p(a_n) < v_p(c) + v_p(a_n)$ so $v_p(c) + v_p(a_n) \le 2v_p(a_{n + 1})$, contradiction.

By lemma $1$ it's also possible to prove that there are finite number of prime $p$ such that $p$ divides at least one $a_n$.

Now consider the following cases:

$\textbf{Case 1.}$ $v_p(a_N) < v_p(c)$.

We must have that $v_p(a_n) \le v_p(c)$ from lemma $1$ and induction.

Suppose there exists $n$ with $v_p(a_{n + 1}) < v_p(a_n)$, then $v_p(a_n) > v_p(a_{n + 1}) \ge v_p(c)$, contradiction.

Therefore, sequence $b_{n} = v_p(a_{n + N})$ is upper bounded and increasing, so it converges.

$\textbf{Case 2.}$ $v_p(a_N) \ge v_p(c)$, then one can prove by induction with lemma $2$ that $v_p(a_n) \ge v_p(c)$.

Suppose there exist $n$ with $v_p(a_{n + 1}) > v_p(a_n)$, then $v_p(a_n) \ge \max(v_p(c),v_p(a_n)) > v_p(a_{n + 1}) > v_p(a_n)$, contradiction.

Therefore, sequence $c_{n}  = v_p(a_{n + N})$ is lower bounded and decreasing, so it converges.

There are finite prime numbers $p$ so the conclusion follows.
This post has been edited 3 times. Last edited by reality95, Jul 11, 2018, 8:29 PM
Z K Y
The post below has been deleted. Click to close.
This post has been deleted. Click here to see post.
orthocentre
72 posts
#18 • 2 Y
Y by Adventure10, buddyram
What is $p$-adic valuation?
This post has been edited 1 time. Last edited by orthocentre, Apr 7, 2019, 9:49 PM
Z K Y
G
H
=
a